At the book store, you purchased some $5 clearance mystery books and $12 regular-priced science fiction books. How many of each did you buy if you spent a total of $126?

Answers

Answer 1

Answer: View answer in explanation below.

Step-by-step explanation: Let's use variables to represent the unknown quantities.

Let x be the number of $5 clearance mystery books purchased.

Let y be the number of $12 regular-priced science fiction books purchased.

We can set up a system of equations based on the given information:

5x + 12y = 126 (total amount spent)

x + y = total number of books purchased

We need to solve for x and y.

Let's use the second equation to solve for one variable in terms of the other:

y = total number of books purchased - x

Now we can substitute this expression for y into the first equation:

5x + 12(total number of books purchased - x) = 126

Simplifying and solving for x:

5x + 12total number of books purchased - 12x = 126

-7x + 12total number of books purchased = 126

-7x = -12total number of books purchased + 126

x = (12total number of books purchased - 126)/7

Since x must be a whole number (you can't buy a fraction of a book), we need to find a value of total number of books purchased that makes x a whole number. We can start by trying different values of total number of books purchased:

If total number of books purchased is 10:

x = (12(10) - 126)/7 = -6/7 (not a whole number)

If total number of books purchased is 11:

x = (12(11) - 126)/7 = 6/7 (not a whole number)

If total number of books purchased is 12:

x = (12(12) - 126)/7 = 6/7 (not a whole number)

If total number of books purchased is 13:

x = (12(13) - 126)/7 = 12/7 (not a whole number)

If total number of books purchased is 14:

x = (12(14) - 126)/7 = 18/7 (not a whole number)

If total number of books purchased is 15:

x = (12(15) - 126)/7 = 24/7 (not a whole number)

If total number of books purchased is 16:

x = (12(16) - 126)/7 = 30/7 (not a whole number)

If total number of books purchased is 17:

x = (12(17) - 126)/7 = 36/7 (not a whole number)

If total number of books purchased is 18:

x = (12(18) - 126)/7 = 42/7 = 6 (a whole number)

So, you bought 6 $5 clearance mystery books and 12 - 6 = 6 $12 regular-priced science fiction books.


Related Questions

The graph of f(t) = 7•2^t shows the value of a rare coin in year t. What is the meaning of the y-intercept?

Answers

Answer:

When it was purchased (year 0) the coin was worth $7

Step-by-step explanation:

we have

[tex]f(t) = 7(2)^t[/tex]

This is a exponential function of the form

[tex]y=a(b)^x[/tex]

where

a is the initial value

b is the base

In this problem we have

[tex]a=\$7[/tex]

[tex]b=2[/tex]

[tex]b=1+r[/tex]

so

[tex]2=1+r[/tex]

[tex]r=1[/tex]

[tex]r=100\%[/tex]

The y-intercept is the value of the function when the value of x is equal to zero

In this problem

The y-intercept is the value of a rare coin when the year t is equal to zero

[tex]f(0)=7(2)^0[/tex]

[tex]f(0)=\$7[/tex]

therefore

The meaning of y-intercept is

When it was purchased (year 0) the coin was worth $7

Answer:

Value of the coin when it was first released

-------------------------------

The y-intercept is the value of f(0).

Substitute t = 0 and find the y-intercept:

f(0) = 7 · 2⁰ = 7 · 1 = 7

This is representing the value of the coin when it was released.

select the acceptable conclusions to a hypothesis test. the value of the test statistic does not lie in the rejection region. therefore, there is insufficent evidence to suggest that the null hypothesis is false. the value of the test statistic lies in the rejection region. therefore, there is sufficient evidence to suggest that the alternative hypothesis is true. the value of the test statistic lies in the rejection region. therefore, there is sufficient evidence to suggest that the null hypothesis is not true. the value of the test statistic does not lie in the rejection region. therefore, there is sufficient evidence to suggest that the null hypothesis is true. the value of the test statistic does not lie in the rejection region. therefore, we accept the null hypothesis.

Answers

The acceptable conclusions of a hypothesis test depend on the test and significance level. If the test statistic falls outside the rejection region, there's insufficient evidence to reject the null hypothesis. If it falls within, there is sufficient evidence to support the alternative hypothesis. The statement is true.

However The acceptable conclusions to a hypothesis test depend on the specific test and the chosen significance level,  in general:

 The value of the test statistic does not lie in the rejection region. Therefore, there is insufficient evidence to suggest that the null hypothesis is false: This statement is correct. If the test statistic falls outside of the rejection region, we fail to reject the null hypothesis at the given significance level. However, this does not mean that the null hypothesis is true, only that we do not have enough evidence to reject it. The value of the test statistic lies in the rejection region. Therefore, there is sufficient evidence to suggest that the null hypothesis is not true: This statement is also correct. If the test statistic falls within the rejection region, we reject the null hypothesis at the given significance level and conclude that the alternative hypothesis is more likely to be true.

Therefore, the acceptable conclusions to a hypothesis test are:

   The value of the test statistic does not lie in the rejection region. Therefore, there is insufficient evidence to suggest that the null hypothesis is false.    The value of the test statistic lies in the rejection region. Therefore, there is sufficient evidence to suggest that the null hypothesis is not true.

The main point of the answer is that the acceptable conclusions to a hypothesis test depend on the specific test and chosen significance level, but generally, if the test statistic falls outside the rejection region, there is insufficient evidence to reject the null hypothesis, and if it falls within the rejection region, there is sufficient evidence to reject the null hypothesis and support the alternative hypothesis.

To learn more alternative hypothesis:

https://brainly.com/question/26185548

#SPJ11

Find the surface area of the rectangular prism.
6 km
4 km
4 km

Answers

Answer: 88km is your answer

Step-by-step explanation: We just had this on a test

Solve for x,
using the tangent lines.
13 cm
X
x = [?] cm Remember: a. b = c. d

Answers

The value of x for the given tangent line is 13 cm.

What is tangent line?

An extended straight line that intersects only one point on a curve and nowhere else is called a tangent. A circle's tangent is always perpendicular to its radius.

Here, two tangents to the same circle, A (x) and B (13), are subtended from two points on the circle.

So, if two tangents are drawn from a point outside the circle, they will both have the same length when they reach the point of contact inside the circle.

The two tangents in this instance have the same outside point of origin. Thus, based on the above calculation, x = 13 cm.

Hence, the value of x for the given tangent line is 13 cm.

Learn more about tangent lines here:

https://brainly.com/question/28385264

#SPJ1

Five people, A,B,C,D, and E want to line up and take a group photo. However, A and B must stand next to each other since they are a couple. Then, what is the total number of ways they can line up?

Answers

In the following question, among the conditions given,To determine the total number of ways five people, A, B, C, D, and E, can line up with the condition that A and B must stand next to each other since they are a couple, we can apply the concept of "permutations." option A, 48, is the correct answer.

Permutations refer to the number of ways that objects can be arranged in a particular order. It is calculated using the formula P(n, r) = n!/(n-r)!, where n represents the total number of objects and r represents the number of objects to be arranged. According to the question, A and B must stand next to each other, so they can be treated as a single entity. Therefore, we have four entities: AB, C, D, and E. We can arrange these four entities in 4! = 24 ways. However, A and B can switch positions among themselves, so each of these 24 arrangements can be arranged in 2 ways. Thus, the total number of ways that five people, A, B, C, D, and E, can line up with the condition that A and B must stand next to each other is 24 × 2 = 48 ways. Therefore, option A, 48, is the correct answer.

For more such questions on permutations

https://brainly.com/question/1216161

#SPJ11

If the measures, in degrees, of the three angles of a triangle, are x, x+10, and 2x-6, the triangle must be:A. RightB. Equilateral,C. ScaleneD. Isosceles

Answers

The triangle is a scalene triangle.

The measures of the three angles of a triangle are x, x+10, and 2x-6. What type of triangle must it be? The sum of the measures of the three angles in a triangle is 180 degrees, which means that:

   x + x + 10 + 2x - 6 = 180

This simplifies to,

   4x + 4 = 180,

   or 4x = 176, or x = 44

Once we have found x, we can now find the measures of the three angles of the triangle: the first angle is x, which is 44°, the second angle is x+10, which is 54°; and the third angle is 2x-6; which is 82°. A scalene triangle is a triangle with all sides and angles of unequal lengths. Since none of the angles has the same measure, the triangle is scalene. The correct answer is option C. Scalene Triangle.

Learn more about types of triangle: https://brainly.com/question/1058720

#SPJ11

8. A department store
buys 300 shirts for
a total cost of $7,200 and sells them for
$30 each. Find the percent markup.

Answers

The percent markup is 25%.

What is percent markup?

Markup percentage is calculated by dividing the gross profit of a unit (its sales price minus it's cost to make or purchase for resale) by the cost of that unit.

Given that, A department store buys 300 shirts for a total cost of $7,200 and sells them for $30 each.

Cost of one shirt [tex]= 7200 \div 300 = \$24[/tex]

And they sold at $30 each,

Percent markup [tex]= 30-24 \div 24 \times 100[/tex]

[tex]= 25\%[/tex]

Hence, the percent markup is 25%.

Learn more about Markup percentage, click:

https://brainly.com/question/28945726

Use Euler's method with step size 0.2 to estimate y(1), where y(x) is the solution of the initial-value problem. (Round your answer to four decimal places.) y' = x^2 + xy y(0) = 4

Answers

By using Euler's method with a step size of 0.2, we estimate that y(1) = 4.5429.

The Euler's method is used to estimate a numerical solution of a first-order differential equation. The formula of Euler's method is given by:

y_1 = y_0 + hf(x_0, y_0)

Where: y_1 is the next value of y after one iterationy

0 is the initial value of yy' = f(x, y)h is the step size

This method is an iterative procedure that advances the estimate of y by one step by approximating the curve using a tangent line at each point along the curve.

Given that y(0) = 4 and h = 0.2, we can use Euler's method to estimate y(1) where y(x) is the solution of the initial-value problemy' = x2 + xy, y(0) = 4

Using Euler's method with a step size of 0.2, we get:

1) When x = 0, y = 4

y_1 = y_0 + hf(x_0, y_0) = 4 + 0.2(0 + 4(0))= 4.02

When x = 0.2, y = 4.02

y_2 = y1 + hf(x_1, y_1) = 4.02 + 0.2(0.2^2 + 0.2(4.02))= 4.10523

When x = 0.4, y = 4.1052

y_3 = y_2 + hf(x_2, y_2) = 4.1052 + 0.2(0.4^2 + 0.4(4.1052))= 4.1994144

When x = 0.6, y = 4.1994

4 = y_3 + hf(x_3, y_3) = 4.1994 + 0.2(0.6^2 + 0.6(4.1994))= 4.3032545

When x = 0.8, y = 4.3033

y_5 = y_4 + hf(x_4, y_4) = 4.3033 + 0.2(0.8^2 + 0.8(4.3033))= 4.4174496

When x = 1, y = 4.4174

y_6 = y_5 + hf(x_5, y_5) = 4.4174 + 0.2(1^2 + 1(4.4174))= 4.5429404

Therefore, using Euler's method with a step size of 0.2, we estimate that y(1) = 4.5429 (rounded to four decimal places).

To know more about Euler's method: https://brainly.com/question/14286413

#SPJ11

Help with geometry with rhombus. Given rhombus ABCD with m

Answers

For the given rhombus the value of x = 3/2 and the measure of angle EAB and angle EBA = 2.5 degrees.

What is a rhombus?

A rhombus is a specific instance of a parallelogram. In a rhombus, opposite sides are parallel and the opposite angles are equal. A rhombus also has equal-length sides on each side, and its diagonals meet at right angles to form its shape. The rhombus is also referred to as a diamond or rhombus. Rhombi or rhombuses are the plural forms of rhombus.

The adjacent angles of a rhombus are supplementary, and diagonals bisect the angles.

Using this property we have:

2(7x - 8) + 2(3x - 2) = 180

14x - 16 + 6x - 4 = 180

20x - 20 = 180

x = 180/20

x = 9

Substituting the value of x:

angle EAB = 7(9) - 8 = 55

angle EBA = 3(9) - 2 = 25

Hence, the value of x = 9 and the measure of angle EAB = 55 degrees and angle EBA = 25 degreed.

Learn more about rhombus here:

https://brainly.com/question/9637480

#SPJ1

Winning the jackpot in a particular lottery requires that you selet the correct four numbers between 1 and 59 and, in a separate drawing, you must also select the correct single number between 1 and 41. Find the probability of winning the jackpot.
The probability of winning the jackpot is __ .

Answers

The probability of selecting the correct four numbers out of 59 is solved by the formula :

P(4 correct numbers) = (number of ways to choose 4 correct numbers) / (total number of possible 4-number combinations)

The total number of possible 4-number combinations out of 59 is:

C(4,59) = (59 choose 4) = 190,578

P(jackpot) = P(4 correct numbers) * P(1 correct number)

P(jackpot) = 1/41

thus, the probability of winning the jackpot in this particular lottery is 1/41.'

To learn about probability visit:

https://brainly.com/question/16484393?referrer=searchResults

#SPJ4

For the year ending December 31, 2017, sales for Company Y were $78.71 billion. Beginning January 1, 2018 Company Y plans to invest 8.5% of their sales amount each year and they expect their sales to increase by 6% each year over the next three years. Company Y invests into an account earning an APR of 1.5% compounded continuously. Assume a continuous income stream. How much money will be in the investment account on December 31, 2020? Round your answer to three decimal places. billion dollars How much money did Company Y invest in the account between January 1, 2018 and December 31, 2020? Round your answer to three decimal places. billion llars How much interest did Company Y earn on this investment between January 1, 2018 and December 31, 2020? Round your answer to three decimal places. If intermediate values are used, be sure to use the unrounded values to determine the answer. billion dollars

Answers

The initial investment (P) is 19.65205 billion dollars. The annual interest rate (r) is 1.5% expressed as a decimal, which is 0.015 and the time period (t) is 3 years. The amount of money in the investment account on December 31, 2020, will be 21.190 billion dollars. Company Y did not invest any additional money into the account between January 1, 2018, and December 31, 2020 and also Company Y earned 1.538 billion dollars in interest on this investment between January 1, 2018, and December 31, 2020.

Now to calculate the amount of money in the investment account on December 31, 2020, we can use the formula for continuous compound interest:

[tex]A = Pe^{(rt)}[/tex]

Where A is the amount of money in the account at the end of the investment period, P is the initial investment, r is the annual interest rate (APR) expressed as a decimal, and t is the time period in years.

First, let's calculate the total amount of money that Company Y plans to invest over the next three years. We know that they plan to invest 8.5% of their sales amount each year, so the total amount invested will be:

[tex]Investment = 0.085 * Sales * (1 + 1.06 + 1.06^2)[/tex]

[tex]Investment = 0.085 * 78.71 * 3.06[/tex]

[tex]Investment = 19.65205[/tex]billion dollars

So, the initial investment (P) is 19.65205 billion dollars. The annual interest rate (r) is 1.5% expressed as a decimal, which is 0.015. The time period (t) is 3 years.

Using the formula, we get:

[tex]A = Pe^{(rt)}[/tex]

[tex]A = 19.65205e^{(0.0153)}[/tex]

[tex]A = 21.190[/tex] billion dollars

Therefore, the amount of money in the investment account on December 31, 2020, will be 21.190 billion dollars.

To calculate the amount of money that Company Y invested between January 1, 2018, and December 31, 2020, we simply subtract the initial investment from the total investment amount:

Amount invested = Investment - P

Amount invested = 19.65205 - 19.65205

Amount invested = 0 billion dollars

Therefore, Company Y did not invest any additional money into the account between January 1, 2018, and December 31, 2020.

To calculate the interest earned on the investment, we simply subtract the initial investment from the amount of money in the account on December 31, 2020:

Interest earned = A - P

Interest earned = 21.190 - 19.65205

Interest earned = 1.53795 billion dollars

Therefore, Company Y earned 1.538 billion dollars in interest on this investment between January 1, 2018, and December 31, 2020.

To practice more questions about initial investment:

https://brainly.com/question/19090357

#SPJ11

I really need help on this question!

Answers

Answer:

The 2nd table

Step-by-step explanation:

For the first set of x and y values, divide y (9/4) by x(3)

Change 3 to 1/3 due to the keep-change-flip rule and multiply that by 9/4 as 9/4 × 1/3= 9/12 which is equivalent to 3/4

Do the same process with 6 and 9/2 and that should give you 3/4 as well

Therefore the second table of values have a proportional relationship to each other as they have the same quotient.

Hope this helps.

PLEASE HELP NOW!!! What would be the experimental probability of drawing a white marble?

Ryan asks 80 people to choose a marble, note the color, and replace the marble in Brianna's bag. Of all random marble selections in this experiment, 34 red, 18 white, 9 black, and 19 green marbles are selected. How does the theoretical probability compare with the experimental probability of drawing a white marble? Lesson 9-3

Answers

The experimental probbaility is 0.225 and the theoretical probability is 0.25

Calculating the experimental probbaility

The experimental probability of drawing a white marble can be calculated by dividing

(1) The number of times a white marble was selected (18)

(2) by the total number of marbles selected (34+18+9+19=80):

So, we have

Experimental probability = 18/80 = 0.225

The theoretical probability

The theoretical probability of drawing a white marble can be calculated by dividing the number of white marbles (1) by the total number of marble colors (4).

So, we have

Theoretical probability = 1/4 = 0.25

This means that the theoretical probability is greater than the experimental probability

Read more about probability

https://brainly.com/question/251701

#SPJ1

(HAZARD) Please help what graph represents Y=-2x+4 I will mark you the brainest

Answers

The graph of Y=-2x+4 should look like a downward sloping line that intersects the y-axis at 4.

What is intersect?

The term "intersect" typically refers to the point or points where two or more things, such as lines, curves, sets, or geometrical shapes, meet or cross each other. The intersection can be described as the common elements or properties shared by the different objects or sets that intersect.

According to question:

The graph of the equation Y=-2x+4 is a straight line with a slope of -2 and a y-intercept of 4. To graph this equation, you can use the slope-intercept form y = mx + b, where m is the slope and b is the y-intercept.

To graph Y=-2x+4, follow these steps:

Plot the y-intercept at (0,4).To locate a different point on the line, use the slope of -2. To do this, move down 2 units and right 1 unit from the y-intercept. This gives you the point (1,2).Between the two points, doodle a straight line.

The graph of Y=-2x+4 should look like a downward sloping line that intersects the y-axis at 4.

To know more about intersect visit:

https://brainly.com/question/14217061

#SPJ1

a grade g1 of 3.50% intersects grade g2 of -2.50% and an equal-tangent curve is desired. a two-way road with a design speed is 65 mph. what is the minimum length the curve must have, to comply with stopping sight distance requirements? (assume you can use the k tables for 0%).

Answers

Grade g1 = 3.50% Grade g2 = -2.50% Design speed = 65 mph. The minimum length the curve must have, to comply with stopping sight distance requirements? The minimum length the curve must have, to comply with stopping sight distance requirements is 196.5 ft or 59.88 m.

`Stopping Sight Distance (SSD) The stopping sight distance (SSD) is the minimum distance a vehicle operator needs to be able to see ahead of the vehicle to bring it to a stop before colliding with an object in its path.The minimum stopping sight distance is given by the following equation: SSD = 0.278Vt + V^2/254f + 1.47W. Where,SSD = stopping sight distance, Vt = total stopping distance, V = design speed, W = width of traveled way, and f = friction factor.To comply with stopping sight distance requirements, the stopping sight distance (SSD) must be equal to or greater than the minimum SSD. K-tables for 0% can be used to determine the minimum SSD. Minimum SSD = SSD min = K x V. Where, SSD min = minimum stopping sight distance, V = design speed, K = adjustment factor from the table. We need to find the minimum length of the curve that meets the stopping sight distance requirements.Here, it is required to design a curve that is the combination of two tangents with an intersection angle of 60° and a length sufficient to maintain an SSD value equal to or greater than the minimum value.Curve length formula:L = (a+b)/sin(θ/2)Where,L = length of curve, a = length of first tangent, b = length of second tangent, and θ = intersection angle L = (a + b) / sin (θ / 2)L = (V^2 / 254f x (Kg1 + Kg2) ) / sin (θ / 2)Length of first tangent, a = V x (1.47 + 0.278Kg1) Length of second tangent, b = V x (1.47 + 0.278Kg2) Intersection angle θ = 60° Friction factor f = 0.35 (for asphalt surface)The adjustment factor from the table for 0% = 0.03. So, we have:Length of first tangent, a = 1.47 x 65 + 0.278 x 65 x 3.5. Length of first tangent, a = 113.1. Length of second tangent, b = 1.47 x 65 + 0.278 x 65 x (-2.5). Length of second tangent, b = 105.9L = (V^2 / 254f x (Kg1 + Kg2) ) / sin (θ / 2)L = (65^2 / (254 x 0.35 x (0.03 x (3.5 + (-2.5))))) / sin (60 / 2)L = 196.5 ft = 59.88 m.

For more questions on stopping sight distance

https://brainly.com/question/2444691

Please help me with the following questions I will give brainiest

Answers

The exponential equation that fits the provided point distribution is [tex]y = 5(2)^x[/tex] . Thus, option A is correct.

What do exponential equations work?

An exponential equation is one in which the exponent contains a variable.

For instance, the exponential equation [tex]y = 5x[/tex] has the variable x as the exponent (also known as "5 to the power of x"),

whereas, the exponential equation y = x5 has the number 5 as the exponent instead of a variable, making the latter equation not exponential.

If we calculate the initial differences, we can determine the exponential equation that corresponds to the given pattern of points as follows:

[tex](2-1) = 5[/tex]

[tex](3-2) = 10[/tex]

[tex](4-3) = 20[/tex]

If we calculate the second differences, we obtain:

[tex](10-5) = 5[/tex]

[tex](20-10) = 10[/tex]

The fact that the second differences are constant shows that the exponential equation's coefficient is 5.

Therefore, The exponential equation that fits the provided point distribution is [tex]y = 5(2)^x[/tex] .

Learn more about exponential here:

https://brainly.com/question/29506679

#SPJ1

a 3-digit pin number is selected. what it the probability that there are no repeated digits? the probability that no numbers are repeated is

Answers

The probability that no numbers are repeated = [tex]\frac{720}{1000}=0.72[/tex]

The probability that there are no repeated digits in a 3-digit pin number is 0.72.

Formula used:

[tex]P(n,r)=\frac{n!}{(n-r)!}\\ Probability=\frac{Number of favourable outcomes}{Total number of events in the samples pace}[/tex]

There are 10 digits (0,1,2,3,4,5,6,7,8,9) to choose from.

Therefore, the total number of possible 3-digit pin numbers with no repeated digits is

[tex]P(10,3)=\frac{10!}{(10-3)!}\\P(10,3)= \frac{10!}{7!}\\P(10,3)=720[/tex]

The total number of possible 3-digit pin numbers [tex]= 10 * 10 * 10 = 1000[/tex].

Thus, the probability that no numbers are repeated = [tex]\frac{720}{1000}=0.72[/tex]

Therefore, the probability that there are no repeated digits in a 3-digit pin number is 0.72.

Learn more about probability below

https://brainly.com/question/13604758

20m:600cm
Reduce the ratios to its simplest forms

Answers

Answer: 10:3

Step-by-step explanation: convert 600cm to 6m, then we will get 20m:6m, 2*10=20 and 2*3=6. then it will become 10:3

HELP PLEASE … Assuming the input of energy continues for another 2.5 seconds, where will the particle be?

A) cannot be determined
B) positive maximum
C) negative maximum
D) equilibrium

Answers

Answer:

To determine the position of a particle given the input of energy, we need to know the type of energy input and the initial position and velocity of the particle. Without this information, we cannot determine the position of the particle after 2.5 seconds.

Therefore, the answer is A) cannot be determined.

Step-by-step explanation:

ABOVE

Evaluate the expression shown below and write your answer as a fraction in simplest form.

-0.25 + 0.3 - ( - 3/10 ) + 1/4

Answers

The evaluation of the expression -0.25 + 0.3 - ( - 3/10 ) + 1/4 is 3 / 5.

How to solve expression?

An algebraic expression is made up of variables and constants, along with algebraic operations such as addition, subtraction, division, multiplication etc.

To evaluate an algebraic expression means to find the value of the expression when the variable is replaced by a given number.

Therefore, let's solve the expression as follows:

-0.25 + 0.3 - ( - 3/10 ) + 1/4

let's convert it to fraction

- 1 / 4 + 3 / 10 + 3 / 10 + 1 / 4

Hence,

3 / 10 + 3 / 10 +  1 / 4  - 1 / 4

3 + 3 / 10

6 / 10 = 3 / 5

learn more on expression here: https://brainly.com/question/24178125

#SPJ1

Use the table to answer the following question. The table shows pairs of values for a linear function. A different function is defined by the equation y = 4x. Which of the following statements is TRUE? The functions both have a slope of 4. The functions are both proportional. The functions have opposite slopes. The functions both have a y-intercept of 4.

Answers

A statement which is true include the following: C. The functions have opposite slopes.

What is the slope-intercept form?

In Mathematics, the slope-intercept form of the equation of a straight line is represented by this mathematical expression;

y = mx + c

Where:

m represent the gradient, slope, or rate of change.x and y represent the data points.c represent the vertical intercept, y-intercept or initial number.

Based on the information provided above, an equation that models one of the function is given by;

y = 4x

mx = 4x

Slope, m = 4.

For the other function, the slope can be calculated as follows;

Slope (m) = (y₂ - y₁)/(x₂ - x₁)

Slope (m) = (0 - 4)/(1 - 0)

Slope (m) = -4/1

Slope (m) = -4.

Read more on slope-intercept here: brainly.com/question/7889446

#SPJ1

what do you mean by arithmetic series?​

Answers

Answer:

The sum of the first n terms in an arithmetic sequence is (n/2)⋅(a₁+aₙ). It is called the arithmetic series formula.

Step-by-step explanation:

An arithmetic series is the sum of the terms in an arithmetic sequence with a definite number of terms. Following is a simple formula for finding the sum: Formula 1: If S nrepresents the sum of an arithmetic sequence with terms , then. This formula requires the values of the first and last terms and the number of terms.


Please mark me as Brainliest if possible! Thank you :)

a college administrator would like to determine how much time students spend on homework assignments during a typical week. a questionnaire is sent to a sample of n=100 student and their response indicates a mean of 7.4 hours per week and standard deviation of 3hours

Answers

Based on the information given, the college administrator has collected a sample of n=100 students and obtained the following statistics: Mean: 7.4 hours per week Standard deviation: 3 hours

Why it is?

These statistics can be used to estimate the average amount of time that all students spend on homework assignments during a typical week, as well as to assess the variability in the data.

To estimate the population mean, the sample mean can be used as an unbiased estimator. This means that the sample mean of 7.4 hours per week is likely a good estimate of the true population mean. However, there is always some uncertainty associated with this estimate due to the fact that it is based on a sample.

To quantify the variability in the data, the standard deviation can be used. A standard deviation of 3 hours indicates that there is a relatively large amount of variability in the amount of time that students spend on homework assignments. Some students may spend significantly more or less time on homework than the average of 7.4 hours per week.

Overall, the college administrator can use this information to gain insights into the amount of time that students spend on homework assignments and to make informed decisions based on this knowledge.

To know more about Statistics related questions, visit:

https://brainly.com/question/29093686

#SPJ1

A package is delivered 3 hours 25 minutes after it is collected, it is collected at 15:39
at what time is the package delivered

Answers

Given the data in the question we calculate that the package is delivered at 18:44.

If the package is collected at 15:39 and delivered 3 hours and 25 minutes later, we can add that amount of time to the collection time to find the delivery time.

First, we need to convert 3 hours and 25 minutes to just minutes. To do this, we multiply 3 by 60 (to convert hours to minutes) and then add 25:

3 hours and 25 minutes = (3 × 60) + 25 = 185 minutes

Now we can add 185 minutes to the collection time of 15:39:

15:39 + 185 minutes = 18:44

Therefore, the package is delivered at 18:44. The delivery time of a package is the time it takes for the package to be transported from the sender to the receiver. In this case, the package was collected at 15:39 and delivered 3 hours and 25 minutes later. To find the delivery time, we added the duration of 3 hours and 25 minutes to the collection time. It is important to keep track of delivery times to ensure timely and efficient shipping, especially for time-sensitive or perishable items. Timely delivery is crucial for businesses that rely on shipping to meet customer expectations and maintain customer satisfaction.

To learn more about amount of time click here

brainly.com/question/29823316

#SPJ4

Is v = [\begin{array}{ccc}1\\1\\3\end{array}\right] an eigenvector of A=[\begin{array}{ccc}1&2&-2\\-2&5&-2\\-6&6&-3\end{array}\right]? If so, find the eigenvalue. Hint: usethe definition of an eigenvalue problem Ax = λx.

Answers

The matrix A with eigenvector v has eigen value equal to -3.

A is the matrix

[tex]A = \left[\begin{array}{ccc}1&2&-2\\-2&5&-2\\-6&6&-3\end{array}\right][/tex]

v is the vector.

[tex]v = \left[\begin{array}{ccc}1\\1\\3\end{array}\right][/tex]

λ is the corresponding eigenvalue

v is an eigenvector of A calculate the corresponding eigenvalue,

Av = λv

Substituting the values of matrix A and eigenvector v , we have,

Calculate left hand side we have,

Av

=

[tex]\left[\begin{array}{ccc}1&2&-2\\-2&5&-2\\-6&6&-3\end{array}\right]. \left[\begin{array}{ccc}1\\1\\3\end{array}\right][/tex]

= [tex]\left[\begin{array}{ccc}-3\\-3\\-9\end{array}\right][/tex]

Now, calculate the right hand side value we have,

λv

=  λ[tex]\left[\begin{array}{ccc}1\\1\\3\end{array}\right][/tex]

= [tex]\left[\begin{array}{ccc}\lambda\\\lambda\\3\lambda\end{array}\right][/tex]

Now, Equate both the sides to get the eigen value ,

λ = -3

Therefore, the eigen value of the matrix A for the eigenvector v is equal to -3.

Learn more about eigenvector here

brainly.com/question/11865376

#SPJ4

The above question is incomplete , the complete question is:

Is [tex]v = \begin{bmatrix}1 \ 3\end{bmatrix}[/tex] an eigen vector of [tex]A = \begin{bmatrix}-1 & 1 \ 6 & 0 \end{bmatrix}\\[/tex] . If so, find the eigenvalue. Hint: use the definition of an eigenvalue problem Ax = λx.

(24p- 10) + (-22p - 2)

Answers

Answer:

2p - 12

Step-by-step explanation:

(24p - 10) + (-22p - 2)

24p - 10 - 22p - 2

2p - 12

So, the answer is 2p - 12

I NEED HELP ON THIS ASAP!!

Answers

In response to the stated question, we may state that x + y < 380 (the variable corporation plans to sell at most 380 boards of wood) (the company expects to sell at most 380 boards of wood)

What is a Variable?

A variable is anything that may be altered in the context of a mathematical notion or experiment. Variables are frequently denoted by a single symbol. The letters x, y, and z are often used as generic variables symbols.

To indicate the amount of boards of each type of wood sold, consider the following variables:

Let x be the number of mahogany boards sold.

Let y represent the number of black walnut boards sold.

The system of inequalities representing the restrictions of this issue scenario may therefore be represented as follows:

[tex]x < 260[/tex] (the corporation has 260 boards of mahogany) (the company has 260 boards of mahogany)

y ≤ 320 (the corporation has 320 boards of black walnut) (the company has 320 boards of black walnut)

x + y < 380 (the corporation plans to sell at most 380 boards of wood) (the company expects to sell at most 380 boards of wood)

Furthermore, we know that the profit on the sale of the wood is $20 per board for mahogany and $6 per board for black walnut. Let P represent the total profit from the sale of the wood. Then: P = 20x + 6y

We may display the three boundary lines x = 260, y = 320, and x + y = 380 to graph this system of inequalities, and shade the feasible region that meets all three inequalities. The triangle bordered by the x-axis, y-axis, and the line x + y = 380 will be the viable region. Here's a basic idea of how the graph may look:

      |

 380 - * - - - - - - - - - - - - - - - - - -

       |  /\                                  

       | /   \                              

       |/     \                            

 260 - *-------* - - - - - - - - - - - - - - -

       0       320

        Mahogany   Black Walnut

To know more about variable visit:

https://brainly.com/question/2466865

#SPJ1

in a large population 54% of the people hav been vaccinated 3 people are randomly selected what is the probability that at least one of them has been vaccinated

Answers

The probability that at least one of the three people has been vaccinated is 92.8%.

Step-by-step explanation: Given, In a large population, 54% of the people have been vaccinated. Then, the probability that one person has been vaccinated is 54/100 = 0.54.

The probability that one person has not been vaccinated is 1 - 0.54 = 0.46. The probability that all three people have not been vaccinated is (0.46)³ = 0.097336. The probability that at least one person has been vaccinated is 1 - 0.097336 = 0.902664. Hence, the probability that at least one of the three people has been vaccinated is 92.8%.

See more about probability at: https://brainly.com/question/13604758

#SPJ11

Decrease R450 in the ratio 9:8​

Answers

Step-by-step explanation:

9+8=17

for ratio 9: 9/17 * 450=R238.24

for ratio 8: 8/17* 450= R211.17

Solution
R450 x 9/17 = R238,24
R450 x 8/17 = R211,76

Megan's standard pay is £17.70 per hour. She gets paid 1.5 times this rate for working overtime. How much will Megan get paid for working 11 hours of overtime? Give your answer to the nearest pound.​

Answers

Answer:

Step-by-step explanation:

Overtime Rate = [tex]17.70 \times 1\frac{1}{2}[/tex]

                        [tex]= 17.70 \times \frac{3}{2}[/tex]

                       [tex]=\frac{53.10}{2}[/tex]

                      [tex]=\pounds26.55[/tex]

Pay [tex]=11*26.55=\pounds 292.05[/tex]    (I assume you can use a calculator for this)

Solution: [tex]\pounds 292[/tex]

                       

Other Questions
what is the role of law enforcement in reporting workplace violence please help me i begging you 1) What symptoms are determined by retropharyngeal abscess at children older than 1 year:A. limited opening mouth, redness of overlaying skin of parotid region.B. drooling, trismus, pain, dysphagia, submandibular mass, and dyspnea or airway compromise caused by displacement of the tongue.C sore throat, neck stiffness, odynophagia, cough.D trismus, pain, odynophagia, drooling, a "hot potato" voice, and fever.E headache, fever, nausea, muscle weakness, tachycardia and other.2) Cutting of abscess taking into account:A. the level of general intoxication and conditions of vital functions of organismB. age and behavior of patient.C. term of onset of antimicrobial therapy according blood cultures.D. his localization, locations of facial nerve, natural folds of skin and conditioning for the optimum outflow of exudation.E surgeon opinion.3) The enzymes for wound treatment are used in case of:A. delay of clearing of wound and tearing away of necrotic tissues.B. intensive exudation from wound.C. significant growing of granulating tissue.D. increased pain in wound after drainage.E. only at children4) Clinical examination of 6-years old child: swelling on submandibular space, painful infiltrate with fluctuant mass in center. Predisposing disease of suppurative inflammation in this age is following:A. nonodontogenic lymphadenitis.B. odontogenic lymphadenitis.C. herpes virus gingivostomatitis.D. acute dento-alveolar abscess.E. acute odontogenic osteomyelitis.5) The skin under swelling on infraorbital area is cyanotic, glossy, pleated does not undertake. Swelling of the eyelids and upper lip, painfull infiltration of infraorbital area on the affected side. Primary diagnosis is following:A. abscess of infraorbital space.B. acute dento-alveolar abscess with involvement of soft tissues.C. acute abscess of cheek.D. phlegmons of space.E. parulis6) The features of abscesses and phlegmons in children are related to the following anatomico-phisiological properties of tissues, except:A. rapid forming of acute purulent hearth (during 2-3 days).B. the intense blood supple of tissues in maxillofacial region.C. hyperpermeability of tissue barrier, that is conductive to distribution of inflammatory process on new tissues structures.D. soft tissues of maxillofacial region in children are characterized by the less closeness of fascies and aponeurosis, which limited anatomic spaces.E. by functional maturity of the lymphatic system. Find the zeros of the function. Then graph the function y= (x+1)(x-2)(x-6) salespeople who can develop a sales proposal that contains specific information about the return on investment are more likely to get a favorable response from key decision makers. (true or false) f an individual buys only two goods and these must be used in a fixed relationship with one another (e.g., coffee and cream), then a. there is no substitution effect from a change in the price of coffee b. there is no income effect from a change in the price of coffee c. Giffen's Paradox must occur if both coffee and cream are inferior goods d. an increase in income will not affect cream purchases an antiport transports sodium into the cells of the pct while pumping hydrogen ions out. what hormone activates this transport? A 12100 kg railroad car is coasting on a level, frictionless track at a speed of 19.0 m/s when a 4790 kg load is dropped onto it.If the load is initially at rest, find the new speed of the car and the % change of the kinetic energy.Hint 1: If the load is dropped into the car, it is like the car is "colliding with a stationary load. If the load is stuck in the car, can they have different final velocities from one another? Both the euglena and paramecium seen here haveAspecialized cells to detect light.Bchlorophyll and are producers.a macronucleus and a micronucleusDprojections that help them move A scientist is studying an organism that is similar to early life on Earth. The scientist observes structures form in the organism that appear as oily spheres with an inner fluid. Of which type of macromolecule is the sphere made? carbohydrate lipid nucleic acid protein ______ is an automated process of analyzing and categorizing social media to determine how many positive, negative, and neutral online comments a brand receives.a. sentiment analysisb. marketing mix Colin wants information about the infrastructure in the countries his company is planning to export to, so he should gather information about the transportation and communications capabilities in each country.True False What would be the effect of a mutation in the gene that encodes RNA polymerase so that it does not bind to the late gene promoter of bacteriophage T4? Solve the equation 50/(1+e^-5r)=25 What is you opinion about people losing jobs(topic coruption) the sentence should be negative which mineral property is associated with breaking on planes? describe the toba eruption, what is the geologic setting, how big was it, where and when did it occur? discuss the duration and changes to climate and how it affected the human population at the time. use wikipedia for this. A dummy activity must have a predefined duration, and it should be added to the total project duration. True or False A brief description of why would help. a leader who demonstrates blank behavior develops trusting relationships with followers, cares for their well-being, and supports them. Who said this quotes "If the world is against the truth, then I am against the world."? Classifying ----- is critical for communicating regional environmental characteristics, observing broad global patterns, and analyzing large amounts of meteorological data collected over years or decades.